Magnetic field along a current-carrying wire (conceptual question)?

In summary, the magnetic field around a point in a current-carrying wire is generated by a bunch of concentric loops in the same plane. If you are outside of the plane of the field, you will not experience it.
  • #1
lillybeans
68
1

Homework Statement



Is the magnetic field around a point in a current-carrying wire radially outward (spherically) like the electric field? Or is it a bunch of concentric circles extending in one plane?

I've always thought of it as a bunch of concentric loops in the same plane, however... After I learned Biot-Savart's law, which is:

bsav2.gif


I wasn't so sure anymore. Why? see diagram

2ir2e6d.jpg


My Question: If the magnetic field around a segment of wire, ds, is a bunch of concentric loops extending in one plane (YZ plane), HOW can point P, which is OUTSIDE of the magnetic field's plane at that point (it's in the XY plane), experience the magnetic field due to a segment far away? Also, how can a segment far away generate a magnetic field at P, like in case 2, where P is not directly above a current-carrying wire?
 
Last edited:
Physics news on Phys.org
  • #2
The magnetic field is found using a cross product. The following is the Biot-Savart's law

B = μ0/(4π) ∫(i.dl×r)/|r3|, where bold denotes "vector".

Let's decompose what this means. Let's pretend that we're only taking the field from a single point contribution. Then we may as well rewrite i.dl, the as q.dv, where q is the charge on a single point particle and dv is the microscopic velocity of this point particle. Makes more sense, right?

B = μ0/(4π) ∫(q.dv×r)/|r3|

The integral is a path integral over the space that the charge moves. Let's consider the point charge and it's velocity at a single time. Hence, let dv = v0 at time t0, so that we can remove the integral. Then

B = μ0/(4π) (q.v0×r)/|r3|

Essentially, this is

B = K.v0×r, where K is a constant and K=μ0/(4π) q/|r3|

Now if you want to visualize this, put it on a coordinate system. Let v0≝(0,0,v0) and calculate the field that comes from this.

Point | Field
(1,0,0) | K.v0(0,1,0)
(1,1,0) | K.v0(-1,1,0)
(1,-1,0) | K.v0(1,1,0)
(0,0,1) | 0

It should be clear now the field is cylindrical with decreasing strength as one leaves the origin. Also, along the z-axis, the field is 0.

What sphere?
 
Last edited:
  • #3
Harrisonized said:
The magnetic field is found using a cross product. The following is the Biot-Savart's law

B = μ0/(4π) ∫(i.dl×r)/|r3|, where bold denotes "vector".

Let's decompose what this means. Let's pretend that we're only taking the field from a single point contribution. Then we may as well rewrite i.dl, the as q.dv, where q is the charge on a single point particle and dv is the microscopic velocity of this point particle. Makes more sense, right?

B = μ0/(4π) ∫(q.dv×r)/|r3|

The integral is a path integral over the space that the charge moves. Let's consider the point charge and it's velocity at a single time. Hence, let dv = v0 at time t0, so that we can remove the integral. Then

B = μ0/(4π) (q.v0×r)/|r3|

Essentially, this is

B = K.v0×r, where K is a constant and K=μ0/(4π) q/|r3|

Now if you want to visualize this, put it on a coordinate system. Let v0≝(0,0,v0) and calculate the field that comes from this.

Point | Field
(1,0,0) | K.v0(0,1,0)
(1,1,0) | K.v0(0,1,-1)
(1,-1,0) | K.v0(0,1,1)
(0,0,1) | 0

It should be clear now the field is a superposition of
1. a field that points toward the charge
2. a field that points around the velocity of the charge (the cylinder-like field that you usually draw around a wire)

Also, along the z-axis, the field is 0.

What sphere?

Thank you for the detailed explanation, though my question remains unanswered. If you look at the diagram I drew, at ds (the red segment), the field only extends outward in one plane. Point P does not experience the field due to this segment simply because it is not in the plane of the field generated. (i.e. none of the green circles I drew will ever touch the blue point, they will always remain parallel to each other) So I don't see how point P can be affected by the field due to any segment that it is not directly below P, since magnetic field is NOT like the electric field, which can expand radially outward in all directions (like a sphere).

So a segment of wire does not only produce a magnetic field immediately surrounding it, but also at a distance?
 
Last edited:
  • #4
I answered your question sufficiently. The field exists everywhere except along the axis of the wire. However, I just realized that I made a mistake in my post earlier. The shape of the field is always cylindrical, and I edited my previous post just now to reflect that.
 
Last edited:
  • #5
Harrisonized said:
I answered your question sufficiently. The field exists everywhere except along the axis of the wire. However, I just realized that I made a mistake in my post earlier. The shape of the field is always cylindrical, and I edited my previous post just now to reflect that.

I see now, thank you very much.
 

Related to Magnetic field along a current-carrying wire (conceptual question)?

1. What is a magnetic field?

A magnetic field is a region in space where a magnetic force can be detected. It is created by moving electric charges, such as those found in a current-carrying wire.

2. How does a current-carrying wire create a magnetic field?

When an electric current flows through a wire, it creates a circular magnetic field around the wire. The direction of the magnetic field can be determined using the right-hand rule, where the thumb points in the direction of the current and the fingers curl in the direction of the magnetic field.

3. Does the strength of the magnetic field depend on the current in the wire?

Yes, the strength of the magnetic field is directly proportional to the current in the wire. This means that the stronger the current, the stronger the magnetic field will be.

4. How does the direction of the current affect the direction of the magnetic field?

The direction of the magnetic field is always perpendicular to the direction of the current. This means that if the current is flowing in a straight line, the magnetic field will form circles around the wire.

5. Can the magnetic field be manipulated by changing the current in the wire?

Yes, the magnetic field can be manipulated by changing the current in the wire. Increasing the current will strengthen the magnetic field, while decreasing the current will weaken it.

Similar threads

Replies
2
Views
151
  • Introductory Physics Homework Help
Replies
16
Views
430
  • Introductory Physics Homework Help
Replies
14
Views
1K
  • Introductory Physics Homework Help
Replies
7
Views
1K
  • Introductory Physics Homework Help
Replies
7
Views
282
  • Introductory Physics Homework Help
2
Replies
43
Views
2K
  • Introductory Physics Homework Help
Replies
11
Views
528
  • Introductory Physics Homework Help
Replies
3
Views
1K
  • Introductory Physics Homework Help
Replies
10
Views
1K
  • Introductory Physics Homework Help
Replies
1
Views
263
Back
Top